What is the Meaning of [J1,J2] = 0 in the Addition of Angular Momenta?

In summary, the condition for angular momentum (vector-)operator is that each component of the operator must commute with each component of the other.
  • #1
tommy01
40
0
hi everybody.

[tex]\textbf{J}_1[/tex] and [tex]\textbf{J}_2[/tex] are angular momentum (vector-)operators.
In many textbooks [tex]\left[\textbf{J}_1,\textbf{J}_2\right] = 0[/tex] is stated to be a condition to show that [tex]\textbf{J}=\textbf{J}_1+\textbf{J}_2[/tex] is also an angular momentum (vector-)operator. But what is meant with [tex]\left[\textbf{J}_1,\textbf{J}_2\right] = 0[/tex]. When i show that [tex]\textbf{J}[/tex] is an angular momentum operator (i.e. [tex]\left[J_x,J_y\right]=iJ_z[/tex] ...) i always need the condition [tex]\left[(\textbf{J}_1)_x,(\textbf{J}_2)_x\right][/tex] and the like. So the components of [tex]\textbf{J}_1[/tex] and [tex]\textbf{J}_2[/tex] should mutually commute. Is this the meaning of [tex]\left[\textbf{J}_1,\textbf{J}_2\right] = 0[/tex]? For me it looks like [tex](\textbf{J}_1)_x(\textbf{J}_2)_x+(\textbf{J}_1)_y(\textbf{J}_2)_y+(\textbf{J}_1)_z(\textbf{J}_2)_z-(\textbf{J}_2)_x(\textbf{J}_1)_x-(\textbf{J}_2)_y(\textbf{J}_1)_y-(\textbf{J}_2)_z(\textbf{J}_1)_z=0[/tex] and this does not imply the conditions i need (as far as i see).
I know Operators acting on different spaces commute and this fact is often used but i want to know how to treat the situation above only with the formal condition [tex]\left[\textbf{J}_1,\textbf{J}_2\right] = 0[/tex].

thanks and greetings tommy.
 
Physics news on Phys.org
  • #2
tommy01 said:
[...] Is this the meaning of [tex]\left[\textbf{J}_1,\textbf{J}_2\right] = 0[/tex]? For me it looks like [tex](\textbf{J}_1)_x(\textbf{J}_2)_x+(\textbf{J}_1)_y(\textbf{J}_2)_y+(\textbf{J}_1)_z(\textbf{J}_2)_z-(\textbf{J}_2)_x(\textbf{J}_1)_x-(\textbf{J}_2)_y(\textbf{J}_1)_y-(\textbf{J}_2)_z(\textbf{J}_1)_z=0[/tex]
Huh? You have two separate copies of the so(3) Lie algebra, so that commutation relation
just says that every generator from copy #1 commutes with every generator from copy #2.

Or did I misunderstand the question?
 
  • #3
Hi. Thank you for your answer. As i mentioned i know that every component of [tex]\textbf{J}_1[/tex] has to commute with every component of [tex]\textbf{J}_2[/tex] to show that [tex]\textbf{J}_1+\textbf{J}_2[/tex] is an angular momentum operator (another generator of the group) and because they act on different subspaces of the system. But the explicit form of [tex]\left[\textbf{J}_1,\textbf{J}_2\right]=0[/tex] (which is the only given condition) is [tex](\textbf{J}_1)_x(\textbf{J}_2)_x+(\textbf{J}_1)_y( \textbf{J}_2)_y+(\textbf{J}_1)_z(\textbf{J}_2)_z-(\textbf{J}_2)_x(\textbf{J}_1)_x-(\textbf{J}_2)_y(\textbf{J}_1)_y-(\textbf{J}_2)_z(\textbf{J}_1)_z=0
[/tex]. So is [tex]\left[\textbf{J}_1,\textbf{J}_2\right]=0[/tex] just an abbreviation for [tex]\left[(\textbf{J}_1)_i,(\textbf{J}_2)_j\right]=0 \forall i,j[/tex] or does the explicit form of the commutator imply this?
greetings.
 
  • #4
tommy01 said:
So is [tex]\left[\textbf{J}_1,\textbf{J}_2\right]=0[/tex] just an abbreviation for [tex]\left[(\textbf{J}_1)_i,(\textbf{J}_2)_j\right]=0 \forall i,j[/tex] or does the explicit form of the commutator imply this?

Yes, the abbrevation you wrote is correct (there is no scalar product involved in the commutator, statement is about each element of the vectors [tex]\mb{J}_1,\mb{J}_2[/tex]). If it helps you can write the elements of what is basically [tex]\mathfrak{su}(2)\otimes\mathfrak{su}(2)[/tex] algebra as [tex](J_1)_i=J_i\otimes 1, [/tex] and [tex](J_2)_i=1\otimes J_i[/tex]. Then it becomes more obvious that the two copies of the underlying algebra commute.

Hope this helps
 
  • #5
many thanks!
 

1. What is the concept of addition of angular momenta?

The addition of angular momenta is a mathematical operation that combines the angular momenta of individual particles or systems to determine the total angular momentum of a larger system. It is used in quantum mechanics to describe the behavior of particles with intrinsic angular momentum, such as electrons, protons, and neutrons.

2. How is the addition of angular momenta calculated?

The addition of angular momenta is calculated using the principles of vector addition. Each individual angular momentum is represented as a vector, with magnitude and direction, and these vectors are added together to obtain the total angular momentum. This process takes into account the spin and orbital angular momenta of the particles or systems being combined.

3. Why is the addition of angular momenta important in quantum mechanics?

The addition of angular momenta is important in quantum mechanics because it allows us to understand and predict the behavior of particles with intrinsic angular momentum. It also helps us to understand the properties of complex systems, such as atoms and molecules, which are made up of multiple particles with different angular momenta.

4. What is the difference between addition of angular momenta and addition of linear momenta?

The addition of angular momenta and addition of linear momenta are similar in that they both involve combining the momenta of individual particles or systems. However, the addition of angular momenta takes into account the direction of the momenta, while the addition of linear momenta only considers the magnitude. Additionally, angular momenta are conserved in a closed system, while linear momenta may change due to external forces.

5. How does the addition of angular momenta affect the energy levels of an atom?

The addition of angular momenta can affect the energy levels of an atom by determining the allowed values of the total angular momentum and its projection onto a specific axis. These values, in turn, determine the energy levels of the atom's electrons, which can impact the atom's properties such as its chemical reactivity and spectral characteristics.

Similar threads

  • Introductory Physics Homework Help
Replies
1
Views
5K
Replies
14
Views
1K
Replies
5
Views
2K
Replies
4
Views
733
  • Quantum Physics
Replies
18
Views
2K
Replies
17
Views
2K
  • Advanced Physics Homework Help
Replies
1
Views
775
Replies
3
Views
1K
  • Quantum Physics
Replies
27
Views
2K
Replies
5
Views
2K
Back
Top